volume 29 • number 8 in this issue · with abdominal pain as a sign of acute gastroenteritis;...

24
Contributors Daniel Park, MD; Keith T. Borg, MD, PhD, FACEP; and William Scott Russell, MD, FACEP , wrote “Imaging in Pediatric Abdominal Pain.” Dr. Park is an assistant professor of pediatrics in the Division of Pediatric Emergency Medicine, and the informatics medical director, Emergency Medicine and Pediatric Emergency Medicine at the Medical University of South Carolina in Charleston. Dr. Borg is director of the Division of Pediatric Emergency Medicine and associate professor of pediatric emergency medicine and emergency medicine at the Medical University of South Carolina in Charleston. Dr. Russell is associate chief medical officer of physician development at Children’s Hospital, medical director of the Division of Pediatric Emergency Medicine, IT medical director, and assistant professor at the Medical University of South Carolina in Charleston. Daniel A. Handel, MD, MPH, FACEP , reviewed “Imaging in Pediatric Abdominal Pain.” Dr. Handel is chief medical officer and an associate professor of medicine and pediatrics in the Division of Emergency Medicine at the Medical University of South Carolina in Charleston. Joy Kay, MD, wrote “Procedural Sedation and Anesthesia.” Dr. Kay is a staff physician at Mercy General Hospital and Methodist Hospital of Sacramento in Sacramento, California. Amal Mattu, MD, FACEP , reviewed “Procedural Sedation and Anesthesia.” Dr. Mattu is a professor and vice chair of the Department of Emergency Medicine and director of faculty development and emergency cardiology fellowships at the University of Maryland School of Medicine in Baltimore. Lynn Roppolo, MD, FACEP , reviewed the questions for these lessons. Dr. Roppolo is an associate professor of emergency medicine and associate emergency medicine residency director at the University of Texas Southwestern Medical Center in Dallas. Louis G. Graff IV, MD, FACEP, is editor-in-chief of Critical Decisions. Dr. Graff is professor of traumatology and emergency medicine at the University of Connecticut School of Medicine in Farmington, Connecticut. Contributor Disclosures. In accordance with the ACCME Standards for Commercial Support and policy of the American College of Emergency Physicians, all individuals with control over CME content (including but not limited to staff, planners, reviewers, and authors) must disclose whether or not they have any relevant financial relationship(s) to learners prior to the start of the activity. These individuals have indicated that they have a relationship which, in the context of their involvement in the CME activity, could be perceived by some as a real or apparent conflict of interest (eg, ownership of stock, grants, honoraria, or consulting fees), but these individuals do not consider that it will influence the CME activity. Sharon E. Mace, MD, FACEP; Baxter Healthcare, consulting fees, fees for non-CME services, and contracted research; Gebauer Company, contracted research; Halozyme, consulting fees. Joshua S. Broder, MD, FACEP; GlaxoSmithKline; his wife is employed by GlaxoSmithKline as a research organic chemist. All remaining individuals with control over CME content have no significant financial interests or relationships to disclose. Method of Participation. This educational activity consists of two lessons with a posttest, evaluation questions, and a pretest; it should take approximately 5 hours to complete. To complete this educational activity as designed, the participant should, in order, take the pretest (posted online following the previous month’s posttest), review the learning objectives, read the lessons as published in the print or online version, and then complete the online posttest and evaluation questions. Release August 1, 2015. Expiration date July 31, 2018. Accreditation Statement. The American College of Emergency Physicians is accredited by the Accreditation Council for Continuing Medical Education to provide continuing medical education for physicians. The American College of Emergency Physicians designates this enduring material for a maximum of 5 AMA PRA Category 1 Credits™. Physicians should claim only the credit commensurate with the extent of their participation in the activity. Each issue of Critical Decisions in Emergency Medicine is approved by ACEP for 5 ACEP Category I credits. Approved by the AOA for 5 Category 2-B credits. A minimum score of 75% is required. Commercial Support. There was no commercial support for this CME activity. Target Audience. This educational activity has been developed for emergency physicians. Volume 29 • Number 8 In This Issue Lesson 15 Imaging in Pediatric Abdominal Pain......................... Page 2 Abdominal pain is a common complaint in pediatric patients, but children are vulnerable to a variety of serious conditions that can present as this seemingly benign complaint. In such cases, emergency physicians must possess a keen diagnostic acumen to prevent major complications and death, as well as a thorough understanding of the appropriate and efficient diagnostic imaging tools to implement, while mitigating the risks of ionizing radiation. Lesson 16 Procedural Sedation and Analgesia ......................... Page 11 Procedural sedation frequently is used in the emergency department for the management of pain and anxiety – a protocol that enables patients to tolerate uncomfortable or unpleasant procedures. Emergency physicians must possess a thorough understanding of how to administer these medications, monitor patients, and manage any complications that might arise. Also in This Issue • The LLSA Literature Review page 9 • The Critical ECG page 10 • The Critical Image page 19 • CME Questions page 22 • The Drug Box page 24 Next Month • Pulmonary Embolism • Implanted Cardiac Devices August 2015

Upload: others

Post on 09-Aug-2020

0 views

Category:

Documents


0 download

TRANSCRIPT

Page 1: Volume 29 • Number 8 In This Issue · with abdominal pain as a sign of acute gastroenteritis; however, appendicitis remains the most common, non-traumatic surgical emergency in

ContributorsDaniel Park, MD; Keith T. Borg, MD, PhD, FACEP; and William Scott Russell, MD, FACEP, wrote “Imaging in Pediatric Abdominal Pain.” Dr. Park is an assistant professor of pediatrics in the Division of Pediatric Emergency Medicine, and the informatics medical director, Emergency Medicine and Pediatric Emergency Medicine at the Medical University of South Carolina in Charleston. Dr. Borg is director of the Division of Pediatric Emergency Medicine and associate professor of pediatric emergency medicine and emergency medicine at the Medical University of South Carolina in Charleston. Dr. Russell is associate chief medical officer of physician development at Children’s Hospital, medical director of the Division of Pediatric Emergency Medicine, IT medical director, and assistant professor at the Medical University of South Carolina in Charleston.

Daniel A. Handel, MD, MPH, FACEP, reviewed “Imaging in Pediatric Abdominal Pain.” Dr. Handel is chief medical officer and an associate professor of medicine and pediatrics in the Division of Emergency Medicine at the Medical University of South Carolina in Charleston.

Joy Kay, MD, wrote “Procedural Sedation and Anesthesia.” Dr. Kay is a staff physician at Mercy General Hospital and Methodist Hospital of Sacramento in Sacramento, California.

Amal Mattu, MD, FACEP, reviewed “Procedural Sedation and Anesthesia.” Dr. Mattu is a professor and vice chair of the Department of Emergency Medicine and director of faculty development and emergency cardiology fellowships at the University of Maryland School of Medicine in Baltimore.

Lynn Roppolo, MD, FACEP, reviewed the questions for these lessons. Dr. Roppolo is an associate professor of emergency medicine and associate emergency medicine residency director at the University of Texas Southwestern Medical Center in Dallas.

Louis G. Graff IV, MD, FACEP, is editor-in-chief of Critical Decisions. Dr. Graff is professor of traumatology and emergency medicine at the University of Connecticut School of Medicine in Farmington, Connecticut.Contributor Disclosures. In accordance with the ACCME Standards for Commercial Support and policy of the American College of Emergency Physicians, all individuals with control over CME content (including but not limited to staff, planners, reviewers, and authors) must disclose whether or not they have any relevant financial relationship(s) to learners prior to the start of the activity. These individuals have indicated that they have a relationship which, in the context of their involvement in the CME activity, could be perceived by some as a real or apparent conflict of interest (eg, ownership of stock, grants, honoraria, or consulting fees), but these individuals do not consider that it will influence the CME activity. Sharon E. Mace, MD, FACEP; Baxter Healthcare, consulting fees, fees for non-CME services, and contracted research; Gebauer Company, contracted research; Halozyme, consulting fees. Joshua S. Broder, MD, FACEP; GlaxoSmithKline; his wife is employed by GlaxoSmithKline as a research organic chemist. All remaining individuals with control over CME content have no significant financial interests or relationships to disclose.

Method of Participation. This educational activity consists of two lessons with a posttest, evaluation questions, and a pretest; it should take approximately 5 hours to complete. To complete this educational activity as designed, the participant should, in order, take the pretest (posted online following the previous month’s posttest), review the learning objectives, read the lessons as published in the print or online version, and then complete the online posttest and evaluation questions. Release August 1, 2015. Expiration date July 31, 2018.

Accreditation Statement. The American College of Emergency Physicians is accredited by the Accreditation Council for Continuing Medical Education to provide continuing medical education for physicians. The American College of Emergency Physicians designates this enduring material for a maximum of 5 AMA PRA Category 1 Credits™. Physicians should claim only the credit commensurate with the extent of their participation in the activity. Each issue of Critical Decisions in Emergency Medicine is approved by ACEP for 5 ACEP Category I credits. Approved by the AOA for 5 Category 2-B credits. A minimum score of 75% is required.

Commercial Support. There was no commercial support for this CME activity.

Target Audience. This educational activity has been developed for emergency physicians.

Volume 29 • Number 8

In This IssueLesson 15 Imaging in Pediatric Abdominal Pain. . . . . . . . . . . . . . . . . . . . . . . . . Page 2

Abdominal pain is a common complaint in pediatric patients, but children are vulnerable to a variety of serious conditions that can present as this seemingly benign complaint. In such cases, emergency physicians must possess a keen diagnostic acumen to prevent major complications and death, as well as a thorough understanding of the appropriate and efficient diagnostic imaging tools to implement, while mitigating the risks of ionizing radiation.

Lesson 16 Procedural Sedation and Analgesia . . . . . . . . . . . . . . . . . . . . . . . . . Page 11Procedural sedation frequently is used in the emergency department for the management of pain and anxiety – a protocol that enables patients to tolerate uncomfortable or unpleasant procedures. Emergency physicians must possess a thorough understanding of how to administer these medications, monitor patients, and manage any complications that might arise.

■ Also in This Issue• The LLSA

Literature Review page 9

• The Critical ECG page 10

• The Critical Image page 19

• CME Questions page 22

• The Drug Box page 24

■ Next Month• Pulmonary Embolism

• Implanted Cardiac Devices

August

2015

Page 2: Volume 29 • Number 8 In This Issue · with abdominal pain as a sign of acute gastroenteritis; however, appendicitis remains the most common, non-traumatic surgical emergency in

Critical Decisions in Emergency Medicine

2

Lesson 15

Imaging in Pediatric Abdominal Pain

Daniel Park, MD; Keith T. Borg, MD, PhD, FACEP; and William Scott Russell, MD, FACEP

■ ObjectivesOn completion of this lesson, you should be able to:

1. Describe common abdominal pathologies that manifest as pain in different pediatric age groups.

2. Determine the appropriate imaging modalities for children presenting with trauma.

3. List the common abdominal masses seen in the pediatric population.

4. Explain the approach to diagnostic imaging in children with chronic abdominal pain.

5. Discuss the approach to diagnostic imaging in postpubertal females.

6. Discuss the risks involved in ionizing radiation in the pediatric population, and describe ways to minimize radiation exposure.

■ From the EM Model

1.0 Signs, Symptoms, and Presentations 1.2.5 Abdominal Pain

Cases

■ Case 1An 8-year-old-boy without a

significant medical history was traveling in the rear seat of a vehicle that struck an oncoming truck while traveling approximately 50 mph. The child was restrained with a lap belt. He is transported via ambulance to your Level 1 trauma center, where is found to be alert and oriented with a GCS of 15. His heart rate is 90 beats per minute, respiratory rate 20 breaths per minute, blood pressure 110/65, and oxygen saturation 100% on room air.

On examination, he remains comfortable in a cervical collar, but complains of tenderness at the mid epigastrium. There is a transverse area of ecchymosis across his abdomen with a minor abrasion over the left lower quadrant. He does not show evidence of any further injury; routine trauma laboratory tests, including hemoglobin and liver function tests, do not reveal any abnormalities.

■ Case 2An 11-year-old girl without a

significant medical history presents with progressively worsening abdominal pain that began 48 hours prior to arrival. She says she has had no appetite today and reports several episodes of non-bloody, non-bilious emesis in the last 48 hours. Her mother reports an oral temperature of 101.3, and says the girl complained of pain during the car ride to the

hospital. The child is alert and oriented and is most comfortable with both hips flexed.

Her heart rate is 125 beats per minute, respiratory rate 18 breaths per minute, blood pressure 120/75, and oxygen saturation 97% on room air. The patient’s eyes are sunken with dry mucous membranes, and there is voluntary guarding on abdominal examination with rebound tenderness at the right lower quadrant. She cries out in pain with a bilateral heel strike. She denies tenderness in the periumbilical, suprapubic, and left portions of her abdomen. Routine laboratory tests reveal a WBC of 21.

CRITICAL DECISIONHow does the age of a child affect the differential diagnosis and subsequent imaging choices?

The evaluation of a pediatric patient with abdominal pain is challenging due, in part, to the evolving nature of the underlying etiologies of pain as the child ages. Classic historical features, including location, timing, character, severity, duration, and radiation of abdominal pain should be considered within the context of a patient’s age.

It can be helpful for emergency physicians to stratify patients into three broad age groups to provide a framework from which a targeted workup should proceed. Distinct intra-abdominal pathologies seem to present in children less than 2 years of age more often than in school-age children and early adolescents.

Page 3: Volume 29 • Number 8 In This Issue · with abdominal pain as a sign of acute gastroenteritis; however, appendicitis remains the most common, non-traumatic surgical emergency in

August 2015 • Volume 29 • Number 8

3

• How does the age of a child affect the differential diagnosis and subsequent imaging choices? 

• How should acute abdominal pain in the setting of trauma be evaluated with imaging in a pediatric patient? 

• How should a potential abdominal mass in a child be approached with diagnostic imaging? 

• What is the best approach to diagnostic imaging in the child with chronic abdominal pain? 

• What imaging considerations should be made when evaluating the postpubertal female patient with abdominal pain? 

• When are CT scans justified, and how can the risks of ionizing radiation be avoided? 

Critical Decisions

However, the astute clinician knows that no age-specific cutoff is absolute; a high index of suspicion should remain for potentially life-threatening etiologies, regardless of age. In any patient less than 2 years old presenting with abdominal pain, the emergency physician should remain wary of conditions such as intussusception, volvulus, and – as always – trauma. Other less common causes, including incarcerated hernia and Hirschprung disease, also should be considered in the very young with non-specific abdominal complaints.

School-age children often present with abdominal pain as a sign of acute gastroenteritis; however, appendicitis remains the most common, non-traumatic surgical emergency in children and should be properly investigated. Abdominal trauma also remains a potential etiology for this age group, as do gynecologic problems in peripubertal females (discussed in a separate section).

IntussusceptionIntussusception, which occurs

most commonly between 3 and 12 months of age, is the leading cause of acute intestinal obstruction in infants.1 Colicky abdominal pain is the classic clinical manifestation of this medical emergency. Although flat and upright plain films of the abdomen can reveal signs of intestinal obstruction and occasionally a characteristic “target” sign, most medical centers have adopted ultrasound evaluation as the first

line for suspected intussusception. Several studies report near 100% sensitivities and specificities of ultrasound in detecting the condition in the pediatric population.2 Ultra-sound also carries the advantage of identifying potential pathologic lead points, as compared with barium or air enema.3 Once the presence of intussusception is confirmed with ultrasound, a therapeutic air enema under fluoroscopic guidance can be performed.

Ultrasound also offers diagnostic benefits without exposing patients to ionizing radiation. As hospitals become more comfortable with this imaging method, its often-cited operator dependency is diminishing. In fact, emergency physicians appear to have the ability to detect intussusception as well as their radiologist colleagues after only 1 hour of focused training in bedside ultrasound.4

Malrotation of the Bowel Malrotation of the bowel is a

congenital condition involving abnormal fixation of the mesentery to the bowel, which predisposes portions of the small and large intestines to volvulize and potentially lose their blood supply. To avoid a medical catastrophe, the emergency physician should maintain a high index of suspicion for this condition in a child with bilious emesis and abdominal pain. The condition usually manifests in utero or during neonatal life, but also can present in

older children.Flat and upright plain radiographs

of the abdomen to investigate for dilated loops of bowel with air-fluid levels is not an unreasonable place to start, but the use of plain x-rays in screening children with abdominal pain has begun to fall out of favor due to its limited diagnostic utility. The study of choice in such cases is an upper GI series. If the diagnosis is confirmed, the patient should be prepared for immediate surgery. A high degree of clinical suspicion must be maintained as the sensitivity of upper GI for malrotation is 94% to 96%.5 For neonates, who are notorious for their inability to localize pain, however, plain radiographs can be a helpful adjunct to a comprehensive history and physical examination to evaluate for intestinal obstruction. X-rays also can detect necrotizing enterocolitis in neonates (most commonly seen in premature births).

Appendicitis In children older than 2 years,

appendicitis and trauma should remain high in the differential diagnosis. Accurate diagnosis and early consultation with a surgeon are important in both scenarios. The child with classic signs and symptoms of appendicitis should be referred to a pediatric surgeon promptly, while avoiding further testing that might delay required surgery. The appropriate path for the investigation of an equivocal case

Page 4: Volume 29 • Number 8 In This Issue · with abdominal pain as a sign of acute gastroenteritis; however, appendicitis remains the most common, non-traumatic surgical emergency in

Critical Decisions in Emergency Medicine

4

can be complicated and potentially confusing. Studies investigating the utility of laboratory tests are myriad, and their application depends on the surgeon. The diagnostic imaging approach has been similarly fraught with debate, but recent literature supports an initial sonographic evaluation followed by abdominal CT scan only after ultrasound has failed to make a definitive diagnosis.

The American College of Emergency Physicians, American Academy of Pediatrics, and American College of Radiology all point to ultrasound as the initial imaging modality of choice. Ultrasound has the advantages of being widely available and relatively inexpensive, while avoiding the dangers of exposing the highly sensitive tissues of pediatric patients to ionizing radiation. The reported sensitivity of ultrasound 80% to 92% with a specificity of 86% to 98%.1 Its use in the evaluation of acute appendicitis, however, has been criticized for the modality’s reliance on operator technique and limited utility in patients with a high body-mass index or retrocecal appendix. The finding of a non-compressible, peristaltic, blind-ending tubular structure arising from the cecum on ultrasound is diagnostic of acute appendicitis.

Although in many hospitals, the CT scan remains the gold standard for diagnosing suspected appendicitis due to its high sensitivity and specificity (>95%) and independence from operator technique, it should be reserved for patients in whom there is a high suspicion for appendicitis and equivocal ultrasound results. Classic CT findings of acute appendicitis include an enlarged appendix, a thickened and enhanced appendiceal wall, and periappendiceal fat stranding.6

CRITICAL DECISIONHow should acute abdominal pain in the setting of trauma be evaluated with imaging in a pediatric patient? 

Blunt TraumaMore than 90% of pediatric

abdominal traumas involve a blunt mechanism.1,7 Children are at increased risk for intra-abdominal complications after blunt injury because of their immature musculoskeletal system and elevated abdominal organ-to-body-mass ratio.1

The initial evaluation of patients suffering from blunt abdominal traumas should focus on the ABCs of the primary survey. The secondary survey and adjuncts to the trauma evaluation should include laboratory and radiologic tests. The utility of a bedside ultrasound FAST scan in adult trauma patients has been well established, with reported sensitivities of 85% to 98%.1 Although the FAST scan’s effectiveness in the pediatric population has not been widely studied, it has been criticized for its insufficient sensitivity and negative predictive value in hemodynamically stable pediatric trauma patients.8

Here again, the effectiveness of the imaging modality appears to be largely dependent upon the skill and experience of the physician.

The CT scan is considered the gold standard for the evaluation of intra-abdominal trauma in pediatrics.1 Its popularity has risen dramatically over the last 15 years9 due to its sensitivity, availability, and ever-decreasing cost. This escalating reliance on CT scans does not come without risk, however, as less than 15% of pediatric patients suffering from blunt abdominal trauma have a documented intra-

abdominal injury, and the majority of these injuries in children are managed nonoperatively.10 As awareness about the risks of exposure to ionizing radiation has grown, a corresponding interest in limiting the “pan-scan” approach to the evaluation of pediatric trauma has developed.

Several leading studies have shown that clinical prediction tools that focus on examination and laboratory data can be used to effectively identify patients at “high risk” of intra-abdominal injury after blunt trauma.7 In fact, clinical prediction models designed to drive selective imaging appear to reduce CT scan use by more than 30%.7,10 A major multicenter prospective study of a prediction tool for intra-abdominal injury focused specifically on physical examination findings revealed a sensitivity of 97% and negative predictive value of 99.9%.11

It is clear that the future of radiologic evaluation in cases of pediatric blunt abdominal trauma likely will show an increased reliance on bedside FAST scans and selective imaging with CT scans for patients identified as being at highest risk for intra-abdominal injury.

Penetrating TraumaAlthough it accounts for less

than 10% of abdominal trauma in children, penetrating trauma can lead to devastating injuries.1

The colon and small bowel are the organs most commonly affected by penetrating injury, followed by the liver, spleen, and major vessels.1 The definitive evaluation and treatment for the unstable patient suffering from penetrating abdominal trauma is exploratory laparotomy. In most

As more data emerges about the harmful

downstream effects of ionizing radiation

on children, clinical practice is compelled

to change as a matter of public health.

Page 5: Volume 29 • Number 8 In This Issue · with abdominal pain as a sign of acute gastroenteritis; however, appendicitis remains the most common, non-traumatic surgical emergency in

August 2015 • Volume 29 • Number 8

5

cases, this procedure should not be delayed to obtain radiologic studies. Local exploration for evidence of peritoneal perforation is imperative in patients who are hemodynamically stable or suffering from superficial anterior or dorsal wounds.1 In stable patients, an abdominal CT scan with IV contrast can be used to confirm the extent of intra-abdominal injury in conjunction with selective laparoscopic evaluation for wounds that penetrate the peritoneum.1

CRITICAL DECISIONHow should a potential abdominal mass in a child be approached with diagnostic imaging?

The evaluation of a potential abdominal mass in a child is informed by the patient’s age. Intra-abdominal masses in neonates tend to originate from the genitourinary tract, whereas malignant lesions such as neuroblastomas, Wilms tumors, and lymphomas predominate in the older child. Initial stabilization of the ill child and a thorough history and physical examination should precede diagnostic imaging.

A multilayered approach, beginning with a plain film of the abdomen, is a reasonable starting point. X-rays can provide useful information about the location of a mass, evidence of an intestinal obstruction, or presence of calcifications that herald a malignant mass such as a teratoma. Ultrasound can provide further information, including the organ of origin and potentially the type of tissue components present in the mass (solid vs. cystic). A CT or MRI scan can reveal even more specific information about the mass, including a more accurate account of its point of origin, other potential areas of involvement, or metastasis in the case of malignancy.

In neonates, hydronephrosis and multidysplastic kidney are the most common etiologies of a palpable abdominal mass.12 Ovarian cysts also can present perinatally; in such cases, sonography is an appropriate

first-line diagnostic imaging test. Though rare, masses that arise from the gastrointestinal tract in neonates often are caused by gastric duplication cysts. Ultrasound can be used to better characterize these lesions, and a Meckel scan can confirm the presence of ectopic gastric tissue frequently found within these masses.13

Neuroblastoma, the most common extracranial solid tumor in childhood, is the most likely cause of an abdominal mass in an infant or older child (approximately 60% to 70% originate from within the abdomen).14,15 Wilms tumor is the second most common etiology of an abdominal mass in childhood and the most common primary renal malignancy. The presenting size of the mass might be rather large, given its retroperitoneal origin. Wilms tumors classically present as painless masses, but can become painful with hemorrhage and rupture. Approximately 15% of children with this disease will have metastatic involvement.16 The lungs, liver, and lymph nodes are the most likely organs of disease spread.

The third most common abdominal mass presenting in childhood is lymphoma. Sixty percent of cases are characterized as non-Hodgkin lymphomas, and a third are intra-abdominal in origin.15 Intussusception often is the presenting sign due to a lymphomatous lead point. These tumors tend to grow rather quickly, but have been known to respond well to chemotherapy.13

In all cases of suspected abdominal mass, ultrasound should be performed first to confirm the lesion; a follow-up CT or MRI scan is warranted to further characterize the organ of origin, determine the extent of involvement, and search for potential areas of metastasis. It is prudent to evaluate the contralateral kidney in the case of a Wilms tumor. Other adjunctive bone, CT, and MRI scans should be tailored to the specific malignancy and individual

patient’s treatment protocol.

CRITICAL DECISIONWhat is the best approach to diagnostic imaging in the child with chronic abdominal pain?

Chronic or recurrent abdominal pain (RAP) is a common presenting complaint in the emergency department. The classic definition of RAP is described as “paroxysmal abdominal pain in children between the ages of 4 and 16 years that persists for more than 3 months and affects normal activity.”17 The Subcommittee on Chronic Abdominal Pain of the North American Society for Pediatric Gastroenterology, Hepatology, and Nutrition offers a more functional definition for RAP: “long-standing, intermittent or constant abdominal pain.”18 Although the prevalence of RAP is unknown, it has been reported in up to 10% to 15% of patients in that age range, and is a common reason for children to miss school.

Functional abdominal pain is far and away the most common diagnosis in patients with RAP.17

Most frequently considered a diagnosis of exclusion, it can lead to expensive and time-consuming evaluations that yield few significant findings.17 Because the emergency department is better suited for the evaluation of acute presentations than chronic symptoms, the very nature of RAP makes it a challenge to manage. The history and physical examination are likely to play a more valuable role in the evaluation of RAP than radiologic tests.

Special attention should be placed on the chronicity of pain, age of onset, associated symptoms, physical examination, and laboratory evaluation. In addition to the management of RAP itself, emergency providers must be attune to the common emergent diagnoses that can occur in patients with chronic pain. They should familiarize themselves with the common “red flags” that can signal an organic etiology of abdominal pain. Pain that wakes a

Page 6: Volume 29 • Number 8 In This Issue · with abdominal pain as a sign of acute gastroenteritis; however, appendicitis remains the most common, non-traumatic surgical emergency in

Critical Decisions in Emergency Medicine

6

patient from sleep or that localizes to a specific area rather than around the umbilicus should lead the emergency physician away from a diagnosis of functional abdominal pain. Functional abdominal pain is not generally associated with significant vomiting or diarrhea; unwanted weight loss; or systemic symptoms such as fever, rash, joint pain, aphthous ulcers, or dysuria. Laboratory findings of guaiac-positive stool, anemia, or elevated inflammatory markers, as well as a family history for inflammatory bowel or peptic ulcer disease, should lead one away from a diagnosis of functional abdominal pain.17,18

The evaluation of chronic abdominal pain is not driven by imaging; therefore, the most effective way to address the radiologic screening of RAP is to discuss the conditions that might lead to the historical or examination “red flags” listed above. The most useful test for the assessment of chronic abdominal pain with obstructive symptoms or dyspepsia (pain with eating, nausea, vomiting, early satiety, and excessive belching) is an upper GI series with small bowel follow through. Depending on the severity of the patient’s presentation, this evaluation might be performed more practically in ambulatory or inpatient settings. In cases of suspected constipation, plain abdominal radiographs can be used to help evaluate the stool load in the colon, but should not supplant a complete physical examination, including a digital rectal examination.

Ultrasound is becoming increasingly popular in the evaluation of abdominal pain in the emergency department, but its role in the evaluation of chronic or recurrent abdominal pain is not as clear. Ultrasound is a good screening tool for particularly concerning symptoms (eg, definitively localized pain), and can help identify acute causes of pain such as appendicitis, cholecystitis, and torsion of the ovary. It also can provide reassurance to the patient and family at the bedside.

Use of an abdominal CT scan is not recommended in cases of chronic or recurrent abdominal pain in the absence of other symptoms such as fever or elevated inflammatory markers.

CRITICAL DECISIONWhat imaging considerations should be made when evaluating the postpubertal female patient with abdominal pain? 

The evaluation of a postpubertal female with abdominal pain can be challenging due to the extensive list of potentially dangerous etiologies. Common diagnoses include dysmenorrhea, mittelschmerz, ruptured ovarian cysts, ovarian torsion, and urinary tract infections. It is incomplete, however, to rest on common culprits if a more serious pathology is possible.

Paramount in the evaluation of such patients is a thorough history and physical examination, including sexual, menstrual, and social histories. A pelvic examination by an experienced practitioner also is an essential element of the workup, and ultrasound examination is warranted in cases suggestive of life-threatening pathology or pregnancy.

PregnancyPostpubertal females who present

to the emergency department with abdominal pain should be considered pregnant until proven otherwise. Urine or serum pregnancy testing (beta-human gonadotropin, beta-hCG) should be considered a default measure in this patient population. The diagnosis of pregnancy can be based on the presence of any of the following: detection of human gonadotropin in the blood or urine, identification of pregnancy by ultrasound examination, or identification of fetal cardiac activity by Doppler ultrasound.

On transvaginal ultrasound examination, a gestational sac usually is visible at 4.5 to 5 weeks of gestation. The yolk sac can be noted at the beginning of the fifth

week of gestation. An embryonic pole with cardiac activity is first detected at 5.5 to 6 weeks.19 A transabdominal approach will allow visualization of these structures at slightly later dates. More advanced ultrasonographic techniques, including biometric measurements, can be used to estimate gestational age and delivery date.

In a known pregnant patient presenting with vaginal bleeding during the first half of pregnancy, consider miscarriage (incomplete, complete, threatened, missed). Bleeding in this situation might or might not be accompanied by abdominal or pelvic pain; thus, any bleeding or pelvic pain in a pregnant woman warrants further evaluation. While a pelvic examination is essential to assess the volume of bleeding and confirm that the uterus is the source, pelvic ultrasound is the most useful test in this situation.20

Ectopic pregnancy is life threatening without medical or surgical intervention. It is the leading cause of death in the first trimester of pregnancy and has seen an increased incidence with the rise in pelvic inflammatory disease (PID) and tubal surgery.21 In a postpubertal female presenting with abdominal pain associated with amenorrhea, the combination of a serum quantitative beta-hCG and transvaginal ultrasound is the most useful diagnostic approach and will provide a definitive diagnosis in almost all cases.19 In normal pregnancies, the serum beta-hCG doubles every 2 days. The hCG concentration rises much more slowly in most, but not all, ectopic and nonviable intrauterine pregnancies.22 Transvaginal ultrasound has a sensitivity of 99% to 100% and a specificity of 80% to 99% for identifying ectopic pregnancy.23

Ovarian TorsionOvarian torsion must be a

diagnostic consideration in any adolescent female presenting with intermittent and often unilateral abdominal or flank pain. Torsion

Page 7: Volume 29 • Number 8 In This Issue · with abdominal pain as a sign of acute gastroenteritis; however, appendicitis remains the most common, non-traumatic surgical emergency in

August 2015 • Volume 29 • Number 8

7

in the premenarchal female also occurs and must be maintained in the differential diagnosis.24 Torsion occurs when the ovary rotates on its pedicle, leading to the compromise of its own blood supply. This is considered a surgical emergency due to the implications of loss of fertility, infection, sepsis, or death. Again, ultrasound is the diagnostic imaging modality of choice; its sensitivity in the diagnosis of ovarian torsion ranges from 46% to 75%.25,26 MRI and CT typically are not used for the evaluation of ovarian torsion, but can be implemented in cases of equivocal ultrasound findings.

Pelvic Inflammatory Disease

Finally, emergency physicians should have a low threshold for the diagnosis of pelvic inflammatory disease (PID) in any adolescent girl presenting with lower abdominal pain and cervical motion tenderness on physical examination. While ultrasound can play a complementary role in the identification of PID or tubo-ovarian abscess, prompt empiric treatment is warranted when clinical suspicion for PID is high.

CRITICAL DECISIONWhen are CT scans justified, and how can the risks of ionizing radiation be avoided? 

Recent literature documents the increasing popularity of CT scans in the pediatric patient population.7 The test can provide invaluable diagnostic information, but comes at the cost of ionizing radiation and uncertain risk. New evidence has determined, beyond theoretical association, that ionizing radiation from CT scans increases the risk of cancer in children.27 Abdominal CT scans deliver approximately 1,000 times the dose of an anteroposterior chest x-ray in adults.28 This dosage calculation is only magnified in the highly dividing tissues of a child.

Risk vs. RewardClinicians should consider the

risks and benefits of radiation dosing when deciding on imaging. Helpful tools such as www.xrayrisk.com can help facilitate these discussions with patients. As more data emerges about the harmful downstream effects of ionizing radiation on children, clinical practice is compelled to change as a matter of public health. Many pediatric medical centers already have begun to reduce their use of CT scans. Of note, a number of hospitals have significantly increased their use of abdominal ultrasound in the evaluation of abdominal pain,29 an uptick that might herald a growing trend.

The future of diagnostic imaging in all patients, including children, will require both the justification and optimization of medical radiation. Justification can be broken down into the “three As”: awareness, appropriateness, and audit.30 In a 2004 survey of radiologists and emergency physicians, approximately 75% significantly underestimated the radiation dose from a CT scan, and a majority did not believe CT scans increased a patient’s lifetime exposure to cancer.31 These sentiments undoubtedly have changed in light of new evidence, but this example highlights the lack of awareness that existed just a few years ago. Radiation safety movements (eg, Image Gently) are furthering the cause by advocating for and educating both providers and patients about the long-term harm of radiation exposure.32

Appropriate use is the second prong of radiation justification. The recent trend of increasing abdominal ultrasound use in the diagnosis of abdominal pain highlights the importance of seeking alternatives to abdominal CT whenever possible. Some medical centers are finding creative ways to limit and optimize radiation exposure. Quick-brain MRI has become more commonplace in the evaluation of potential brain injury; other hospitals are adopting low-dose CT scans to keep medical radiation as

Pearls• Ultrasound is the initial

imaging study of choice in the emergency department evaluation of a patient with potential intussusception.

• Children with a history and physical examination findings suggestive of classic appendicitis (ie, periumbilical pain radiating to the RLQ, anorexia, fever, emesis) do not require radiographic studies to confirm the diagnosis.

• Neuroblastoma is the most common malignancy presenting as abdominal mass in an infant or child.

• Rule out pregnancy in any postpubertal female patient presenting with abdominal pain.

Pitfalls• Failing to obtain a thorough

history (eg, sexual, menstrual, and social) and fully evaluate a postpubertal female patient with abdominal pain.

• Relying on bloodwork or imaging tests to diagnose functional abdominal pain. This is a diagnosis of exclusion, and is not associated with radiographic abnormalities or elevated inflammatory markers.

• Unnecessarily exposing pediatric patients to ionizing radiation through CT scans. The risks posed to this vulnerable population, including the latent development of cancer, must be considered.

Page 8: Volume 29 • Number 8 In This Issue · with abdominal pain as a sign of acute gastroenteritis; however, appendicitis remains the most common, non-traumatic surgical emergency in

Critical Decisions in Emergency Medicine

8

low as reasonably achievable.The most effective way to reduce

the use of CT scans, however, is to avoid them altogether. Research indicates that CT scans could be replaced by alternative modalities or not performed at all in up to one-third of cases.33

Case Resolutions

■ Case 1After a complete primary survey,

including a negative FAST (focused assessment with sonography for trauma) examination, the car accident victim underwent an abdominal CT scan with contrast that revealed a duodenal hematoma. He was admitted to the pediatric surgery service, and total parenteral nutrition and nasogastric decompression were initiated. He remained in the hospital for several weeks for conservative management, and was discharged with resolution of the duodenal hematoma on follow-up abdominal CT scan. This case illustrates the importance of maintaining a high index of suspicion for intra-abdominal injury in scenarios involving severe blunt trauma to the abdomen.

■ Case 2The pediatric surgery service

was consulted promptly; given the 11-year-old girl’s constellation of symptoms, the decision was made to operate the following morning. Intravenous isotonic fluids, analgesia, and antibiotics were initiated and she underwent a laparoscopic appendectomy without complications. She was discharged from the hospital the following day in stable condition. This case illustrates that not all children in whom there is a high index of suspicion for appendicitis require imaging. If the clinical picture is unclear, abdominal ultrasound has become the preferred initial diagnostic modality in this patient population. If ultrasound results are equivocal and the clinical scenario warrants further investigation, abdominal CT can be a reasonable diagnostic tool.

Summary The approach to imaging in the

pediatric population is continually evolving as more literature concerning long-term radiation risks is revealed and newer technology is implemented. However, the principles of emergent stabilization of the child with abdominal pain remain the same. A complete assessment of the patient’s airway, breathing, and circulation is paramount. A thorough history and physical examination with an awareness of intra-abdominal pathologies specific to certain age groups will provide important diagnostic information. The remainder of the workup relies on the judicious use of imaging to arrive at a timely diagnosis.

References 1. Fleisher and Ludwig. Abdominal Pain in Textbook of

Pediatric Emergency Medicine. Lippincott Williams & Wilkins; Sixth edition (2010)

2. Manson D. Contemporary imaging of the child with abdominal pain or distress. Paediatr Child Health. 2004; 9(2):93–97.

3. Navarro O, Dugougeat F, Kornecki A, et al. The impact of imaging in the management of intussusception owing to pathologic lead points in children. A review of 43 cases. Pediatr Radiol. 2000;30(9):594–603.

4. Riera A, Hsiao AL, Langhan ML, et al. Diagnosis of intussusception by physician novice sonographers in the emergency department. Ann Emerg Med. 2012;60(3):264–268.

5. Applegate KE, Anderson JM, Klatte EC. Intestinal malrotation in children: a problem-solving approach to the upper gastrointestinal series. Radiographics 2006;26(5):1485–1500.

6. Leung AK, Sigalet D. Acute abdominal pain in children. American Family Physician. 2003;67(11).

7. Holmes JF, Mao A, Awasthi S, et al. Validation of a prediction rule for the identification of children with intra-abdominal injuries after blunt torso trauma. Ann Emerg Medicine. 2009;54(4):528–533.

8. Coley BD, Mutabagani KH, Martin LC, et al. Focused abdominal sonography for trauma (FAST) in children with blunt abdominal trauma. J Trauma. 2000;48(5):902–906.

9. Larson DB, Johnson LW, Schnell BM, et al. Rising use of CT in child visits to the emergency department in the United States, 1995–2008. Radiology. 2011;259(3):793–801.

10. Streck CJ Jr, Jewett BM, Wahlquist AH, et al. Evaluation for intra-abdominal injury in children after blunt torso trauma: can we reduce unnecessary abdominal computed tomography by utilizing a clinical prediction model? J Trauma Acute Care Surg. 2012;73(2):371–376.

11. Holmes JF, Lillis K, Monroe D, et al. Identifying children at very low risk of clinically important blunt abdominal injuries. Ann Emerg Med. 2013;62(2):107–116.

12. Farmer DL. Urinary tract masses. Semin Pediatr Surg. 2000;9(3):109–114.

13. Rahhal RM, Eddine AC, Bishop WP. A child with an abdominal mass. Hospital Physician. 2006.

14. Goodman MT, et al. In: Ries L, et al. Cancer incidence and survival among children and adolescents: United States SEER program 1975–1995. Bethesda, Maryland: National Cancer Institute; 1999:65-72. NIH Pub. No. 99-4649.

15. Golden CB, Feusner JH. Malignant abdominal masses in children: quick guide to evaluation and diagnosis. Pediatr Clin North Am. 2002;49(6):1369–1392, viii.

16. Breslow NE, Churchill G, Nesmith B, et al. Clinicopathologic features and prognosis for Wilms’ tumor patients with metastases at diagnosis. Cancer. 1986;58(11):2501–2511.

17. Boyle JT. Recurrent abdominal pain: an update. Pediatri Rev. 1997;18(9):310–320.

18. American Academy of Pediatrics Subcommittee on Chronic Abdominal Pain. Chronic abdominal pain in children. Pediatrics. 2005;115(3):812–815

19. Brown K. Evaluation of acute pelvic pain in the adolescent female. Available at: Up to Date.com. Accessed November 21, 2013.

20. Strobino B, Pantel-Silverman J. Gestational vaginal bleeding and pregnancy outcome. Am J Epidemiol. 1989;129(4):806–815.

21. Jehle D, Krause R, Braen GR. Ectopic pregnancy. Emerg Med Clin North Am. 1994;12(1):55–71; McDonald JA, et al. Abdominal pain in the adolescent female. Clin Ped Emerg Med. 2002;3:33–44.

22. Silva C, Sammel MD, Zhou L, et al. Human chorionic gonadotropin profile for women with ectopic pregnancy. Obstet Gynecol. 2006;107(2):605–610.

23. Counselman FL, Shaar GS, Heller RA, King DK. Quantitative B=hCG levels less than 1000 mIU/mL in patients with ectopic pregnancy: pelvic ultrasound still useful. J Emerg Med. 1998;16(5):699–703.

24. Cass DL. Ovarian torsion. Semin Pediatr Surg. 2005;14(2):86–92.

25. Wilkinson C, Sanderson A. Adnexal torsion—a multimodality imaging review. Clin Radiol. 2012;67(5):476–483.

26. Mashiach R, Melamed N, Gilad N, et al. Sonographic diagnosis of ovarian torsion: accuracy and predictive factors. J Ultrasound Med. 2011;30(9):1205–1210.

27. Pearce MS, Salotti JA, Little MP, et al. Radiation exposure from CT scans in childhood and subsequent risk of leukaemia and brain tumours: a retrospective cohort study. Lancet. 2012;380(9840):499–505.

28. Brenner DJ, Hall EJ. Computed tomography—an increasing source of radiation exposure. N Engl J Med. 2007;357(22):2277–2284.

29. Menoch MJ, Hirsh DA, Khan NS, et al. Trends in computed tomography utilization in the pediatric emergency department. Pediatrics. 2012;129(3):e690–e697.

30. Malone J, Guleria R, Craven C, et al. Justification of diagnostic medical exposures: some practical issues. Report of an international atomic energy agency consultation. Br J Radiol. 2012;85(1013):523–538.

31. Lee CI, Haims AH, Monico EP, et al. Diagnostic CT scans: assessment of patient, physician, and radiologist awareness of radiation dose and possible risks. Radiology. 2004;231(2):393–398.

32. Referring Providers—Nuclear Medicine. Imagegently.org. Retrieved January 9, 2014, from http://www.imagegently.org/Procedures/NuclearMedicine/ReferringProvidersNuc.asp

33. Slovis TL, Berdon WE. Panel Discussion. Pediatr Radiol. 2002;32:242–244.

Page 9: Volume 29 • Number 8 In This Issue · with abdominal pain as a sign of acute gastroenteritis; however, appendicitis remains the most common, non-traumatic surgical emergency in

August 2015 • Volume 29 • Number 8

9

Wafarin inhibits the synthesis of vitamin K-dependent coagulation factors II, VII,

IX, and X. Reversal of warfarin in the setting of life-threatening hemorrhage traditionally is done with vitamin K and fresh frozen plasma (FFP). However, this method can be time consuming and requires substantial fluid administration.

Vitamin K replacement restores the production of intrinsic clotting factors necessary for sustained reversal of anticoagulation. The recommended dose is 5 to 10 mg intravenously; however, it can take up to 4 hours to achieve the desired effects. The rate of anaphylaxis is only 3/10,000.

Fresh frozen plasma requires ABO blood group compatibility testing and 30 to 60 minutes to thaw. The initial recommended minimal dose for warfarin anticoagulation reversal is 15 ml/kg (approximately 4 units for the average 70-kg person); it typically takes 13 to 48 hours to achieve full effect.

Recombinant-activated factor VIIa (rFVIIa) has been used as an off-label treatment for hemorrhage in non-hemophiliac patients. In healthy patients with an international normalized ratio (INR) less than 2, the agent has been proven to reverse the INR in less than 1 hour; however, the clinical impact of this reversal is unknown. rFVIIa carries a risk of thrombotic events (10% to 20%), but should remain a consideration in patients with religious objections to blood products.

Prothrombin complex concentrates (PCCs)

are derived from pooled human plasma. Three-factor PCCs contain factors II, IX, and X, proteins C and S, and a small amount of heparin; 4-factor PCCs also contain factor VII. These agents require a small volume of administration (<100 mL) and eliminate the need for ABO-compatibility testing. Reversal of INR can occur in 10 to 30 minutes and lasts for about 6 hours. The use of PCCs in brain hemorrhage has been associated with improved neurologic outcomes and reduced hematoma growth, compared to FFP. Risks include thrombotic events (occurring in 0.9% to 3.8% of patients) and the potential for transmitting infectious agents.

The LLSA Literature Review“The LLSA Literature Review” summarizes articles from ABEM’s 2016 Lifelong Learning and Self-Assessment Reading List. These articles are available online in the ACEP LLSA Resource Center (www.acep.org/llsa) and on the ABEM website.

Rapid Reversal of Warfarin-Associated Hemorrhage in the Emergency Department by Prothrombin Complex Concentrates

Article 1

Key Points• For warfarin reversal of life-threatening

bleeding, vitamin K should be administered early through an IV route.

• FFP should be given when other agents are unavailable.

• PCC, preferably 4-factor, is an attractive option for wafarin reversal due to its low volume, faster INR reversal, and growing evidence of clinical superiority.

• When using 3-factor PCC, consider using FFP or rFVIIa, given the lack of factor VII.

Reviewed by David Yamane, MD and J. Stephen Bohan, MS, MD, FACEP; Harvard Affiliated Emergency Medicine Residency, Brigham and Women’s Hospital

Frumkin K. Rapid reversal of warfarin-associated hemorrhage in the emergency department by prothrombin complex concentrates. Ann Emerg Med. 2013;62(6):616–26.

Page 10: Volume 29 • Number 8 In This Issue · with abdominal pain as a sign of acute gastroenteritis; however, appendicitis remains the most common, non-traumatic surgical emergency in

Critical Decisions in Emergency Medicine

10

The Critical ECG■ Case A 26-year-old man presents with severe weakness and lightheadedness.

Sinus bradycardia (SB), rate 45, frequent premature atrial complexes (PACs) in a pattern of quadrigeminy, peaked T waves suggestive of hyperkalemia. The overall rhythm is regularly irregular. Regular irregularity should suggest one of two possibilities: second-degree atrioventricular (AV) block or regular-occurring PACs. Second-degree AV block is ruled out because all of the P waves are conducted. On the other hand, PACs appear every fourth beat and are followed by a pause in the regular cycle of beats. Peaked T waves are noted in the lateral precordial leads as well, diagnostic of hyperkalemia. Hyperkalemia-associated T waves can be distinguished from other causes of large T waves by their narrow base, symmetric shape, and “sharp” peaked apex. Other causes of large T waves tend to be broad-based and more rounded at the apex. The patient’s serum potassium level was 8.3 mEq/L. Although common teaching regarding ECG manifestations of hyperkalemia focuses on peaked T waves, widening of the QRS complex, and ventricular dysrhythmias, hyperkalemia is well known to produce unusual bradycardias, AV blocks, fascicular blocks, and bundle branch blocks as well.

Feature Editor: Amal Mattu, MD, FACEP. From: Mattu A, Brady W. ECGs for the Emergency Physician 2. London: BMJ Publishing; 2008:11,23. Reprinted with permission.

Page 11: Volume 29 • Number 8 In This Issue · with abdominal pain as a sign of acute gastroenteritis; however, appendicitis remains the most common, non-traumatic surgical emergency in

August 2015 • Volume 29 • Number 8

11

Lesson 16

Procedural Sedation and Analgesia

Joy Kay, MD

■ ObjectivesOn completion of this lesson, you should be able to:

1. Explain the sedation continuum and define the terms “moderate,” “deep,” and “dissociative” sedation.

2. Describe the indications for procedural sedation.

3. Describe the pharmacology of commonly used agents and their associated adverse events.

4. List the absolute and relative contraindications for ketamine use as a procedural sedation agent.

5. Describe the personnel, equipment, and monitoring protocols needed to perform procedural sedation.

■ From the EM Model

19.0 Procedures and Skills Integral to the Practice of Emergency Medicine 19.3.3 Procedural sedation and analgesia

Procedural sedation and analgesia (PSA) is defined by the American College of Emergency Physicians (ACEP) as a technique for administering sedatives or dissociative agents with or without analgesics to induce an altered state of consciousness.1 PSA allows patients to tolerate unpleasant procedures while maintaining cardiorespiratory function. It often is used in the emergency department for painful procedures such as joint reductions, cardioversions, and laceration repairs (Table 1). While the use of PSA is common, the practice varies across institutions.

Case Presentations

■ Case 1A 3-year-old boy presents with

an actively bleeding laceration on his forehead. His parents report that he was at home playing with an older sibling when he fell, striking

his head on the edge of a table. He cried immediately following the accident and never vomited or lost consciousness. The patient is otherwise healthy, and his immunizations are up to date. On physical examination, he is awake and alert and acting appropriately for his age. There is a deep, linear laceration (4 cm long) over the patient’s right eyebrow that continues to bleed. The boy is apprehensive about being approached, and will not allow a proper examination to be done.

■ Case 2A 47-year-old man presents with a

complaint of palpitations that started suddenly approximately 1 hour ago while he was sitting at his work desk. He denies any chest pain, but states that he feels short of breath and lightheaded. The patient does not have a significant medical history, but regularly consumes alcohol (roughly 2 beers per day). Upon arrival, initial vital signs are pulse 140, blood pressure 90/45, respiratory rate 22, and oxygen saturation 97%. An EKG shows an irregularly irregular rhythm with narrow QRS complexes. Aside from the patient’s irregular heartbeat, the physical examination is normal. A peripheral IV is established and the patient is placed on a cardiac monitor.

■ Case 3A 72-year-old man is brought

in by EMS after a highway motor vehicle accident in which he was the

Table 1.Indications for PSA7

• Cardioversion

• Joint/fracture reduction

• Incision and drainage

• Wound debridements

• Laceration repair

• Lumbar puncture

• Chest tubes/central lines

Page 12: Volume 29 • Number 8 In This Issue · with abdominal pain as a sign of acute gastroenteritis; however, appendicitis remains the most common, non-traumatic surgical emergency in

Critical Decisions in Emergency Medicine

12

• What pertinent information must be obtained in the pre-sedation evaluation of a patient?

• What are the biggest advantages and disadvantages of commonly used PSA medications?

• What clinical personnel are required when performing procedural sedation?

• How should patients be monitored during PSA, and what equipment is necessary?

• When is it safe to discharge a patient home following PSA?

Critical Decisions

front passenger of a car that struck another vehicle from behind. He was wearing a seatbelt and the air bags deployed. The patient denies any loss of consciousness or head trauma, but complains of extreme right hip and leg pain. Initial vital signs are pulse 102, blood pressure 138/78, respiratory rate 18, and oxygen saturation 96%. On physical examination, you record a GCS of 15 with equal breath sounds and a non-tender abdomen. The patient indicates pain on palpation of the right hip, and his right leg is internally rotated and shortened compared to left; he has intact dorsalis pedis bilaterally.

DefinitionsProcedural sedation exists

in a continuum of altered level of consciousness ranging from minimal to general anesthesia, with dissociative sedation falling outside of the continuum. Minimal sedation (anxiolysis) is defined as a drug-induced state during which patients respond normally to verbal commands, while their ventilatory and cardiovascular functions remain unaffected. Moderate sedation is defined as a drug-induced depression of consciousness during which patients respond purposefully to verbal commands or light stimulation, while spontaneous ventilation and cardiovascular function are maintained. Deep sedation is defined as a drug-induced depression of consciousness during which patients cannot be easily aroused but respond purposefully to repeated or painful

stimuli. Deep sedation causes possible impairment in ventilation, although cardiovascular function is maintained. General anesthesia is defined as a drug-induced loss of consciousness during which patients are not arousable and may have an impaired cardiorespiratory function that requires support. Dissociative sedation is defined as a trance-like cataleptic state induced by ketamine. It is characterized by profound analgesia and amnesia with maintenance of airway reflexes, spontaneous respirations, and cardiopulmonary stability.2,5

CRITICAL DECISIONWhat pertinent information must be obtained in the pre-sedation evaluation of a patient?

Prior to starting procedural sedation in any patient, a focused history and physical examination must be performed. A careful history should include diseases involving major organs, current medications, allergies, past experience with sedation agents, and most recent oral intake. A focused physical examination should include a thorough airway assessment, as well as heart and lung examinations.3 Consider using the Mallampati classification or LEMON mnemonic (look, evaluate, Mallampati class, obstruction, neck) to predict difficult intubation during airway assessment. Routine laboratory testing is not required and should be guided by the patient’s underlying medical conditions. As with all procedures

done in the emergency department, the risks, benefits and potential complications must be discussed with the patient, and consent must be obtained.3,4

Recent food intake alone is not a contraindication to PSA, but aspiration risk should be determined according to each individual patient’s risk factors, timing of last meal, and urgency of the procedure.5 Risk factors for aspiration include older age, underlying medical conditions, airway difficulties, and conditions predisposing patients to gastroesophageal reflux such as hiatal hernia, bowel obstruction, ileus, and peptic ulcer disease.3

The American Society of Anesthesiologists recommends at least 2 hours of fasting for clear liquids; 4 hours for breast milk; and 6 hours for solids, cow’s milk, or infant formula (Table 2). These guidelines are based on healthy patients undergoing elective operative procedures, and are

Table 2.Recommended Fasting Guidelines by the American Society of Anesthesiologists3

Type of Food Fasting Period

Clear liquids 2 hours

Breast milk 4 hours

Infant formula 6 hours

Milk 6 hours

Light meal/solids

6 hours

Page 13: Volume 29 • Number 8 In This Issue · with abdominal pain as a sign of acute gastroenteritis; however, appendicitis remains the most common, non-traumatic surgical emergency in

August 2015 • Volume 29 • Number 8

13

not necessarily applicable to patients undergoing procedural sedation in the emergency department. In fact, multiple studies of emergency department procedural sedation show no evidence that fasting has any impact on aspiration risk.6,2 There also is no evidence to support the idea that prophylaxis with antacids, histamine-2 blockers, or anticholinergics lowers aspiration risk or improves outcomes.

CRITICAL DECISIONWhat are the biggest advantages and disadvantages of commonly used PSA medications?

There are many medications in different drug classes that can be used alone or in combination for procedural sedation and analgesia. No one drug is appropriate for use in every situation. It is up to the emergency physician to select the best agent after considering its risks and benefits, and the type and length of procedure to be performed. Among the most commonly used medications for PSA in the emergency department are ketamine, propofol, ketamine/propofol, versed/fentanyl, etomidate, and dexmedetomidine.

KetamineKetamine is a phencyclidine

derivative with dissociative, sedative, analgesic, and amnestic properties;7 it preserves airway reflexes and allows spontaneous respirations.8 Ketamine non-competitively blocks the N-methyl-D-aspartate receptor, decreasing the excitatory neurotransmission and causing a functional dissociation between the cortical and limbic systems of the brain.9,7 It also is a weak bronchodilator that possesses some opioid and sympathomimetic properties. As a result, ketamine has analgesic effects and can cause an elevation in heart rate, blood pressure, and myocardial demand.10

Unlike other agents used in PSA, ketamine’s effect on sedation is not dose dependent. Ketamine

dissociation appears at approximately 1-1.5 mg/kg intravenously (IV) and 3-4 mg/kg intramuscularly (IM). Once the dissociation is reached, administration of additional ketamine does not deepen the sedation.9 The current recommended regimen by IV route is an initial dose of 1.5-2 mg/kg in children or 1 mg/kg in adults, with incremental doses of 0.5-1 mg/kg every 5-10 minutes. The recommended IM regimen is 4-5 mg/kg followed by incremental doses of 2-4 mg/kg.

Both IM and IV routes display similar rates of respiratory and adverse agitation effects.9 While the IM route is useful when IV access cannot be achieved, its disadvantages include longer recovery times and higher rates of emesis. Current ACEP guidelines do not recommend the IM route for adult patients.

Ketamine has been used safely in the pediatric population since its discovery in the 1960s. However, its use in adults remains more limited due to an increased risk of emergent reactions, which occur more frequently in adults than in children. Other notable side effects of ketamine include laryngospasm, hypersalivation, respiratory depression, apnea (mostly transient),

and emesis (most common in early adolescence and with an IM route).9

Emergent reactions (reported in 7.6% of pediatric and 10% to 20% of adult patients10) are well documented and include increased agitation, restlessness, dysphoria, hallucinations, and nightmares.10,11 There is some research to suggest

a significant reduction in recovery agitation when midazolam prophylaxis is given with ketamine.3 Other studies, however, conclude that this approach fails to reduce the occurrence of emergent reactions, but does decrease the incidence of emesis.2 Because the co-administration of midazolam increases the risk of respiratory complications, the current recommendation in both children and adults is to use it only when needed.

Laryngospasm is a rare but potentially life-threatening complication that occurs in roughly 0.4% of patients.10 Laryngospasm is caused by the closure of true vocal cords, leading to partial or complete airway obstruction. This complication is more common in children than in adults; it occurs shortly after drug administration and usually is transient.7 Despite the potential for disastrous airway problems, most cases of laryngospasm resolve with simple airway maneuvers or with bag-valve mask ventilation; endotracheal intubation is rarely necessary.

Hypersalivation is another known side effect of ketamine, prompting some to prophylactically use anticholinergic agents such as atropine or glycopyrrolate to decrease the risk of aspiration and respiratory complications. Because anticholinergic agents are associated with a higher incidence of airway complications, ACEP guidelines do not support their routine use with ketamine.

Absolute contraindications to

Proper equipment and supplies must be in

place to manage the potential complications

of PSA, including allergic reactions,

cardiac arrest, and respiratory arrest.

Page 14: Volume 29 • Number 8 In This Issue · with abdominal pain as a sign of acute gastroenteritis; however, appendicitis remains the most common, non-traumatic surgical emergency in

Critical Decisions in Emergency Medicine

14

ketamine use are an age less than 3 months (due to an increased rate of airway complications) and a history of/suspected psychosis (due to potential exacerbation of the condition). Relative contraindications include procedures involving stimulation of the posterior oropharynx, underlying cardiovascular disease, and underlying thyroid disorders.9 Conditions leading to increased intracranial pressure (eg, intracranial mass, hydrocephalus) and increased intraocular pressure (eg, acute globe injury, glaucoma) are regarded as relative contraindications.9 More studies have recently emerged, however, that appear to refute this finding (Table 3).

Propofol Propofol is a highly lipid-soluble,

sedative-hypnotic drug that has no analgesic effects. It appears to operate by potentiating the gamma-aminobutyric acid receptor system. The drug possesses many properties that are desirable for procedural sedation, including a rapid onset and extremely short half-life, which results in few residual effects once it wears off.12 Because it is lipid soluble, propofol is rapidly and extensively distributed in the body; it crosses the blood-brain barrier, resulting in a quick onset time of 30-45 seconds. Propofol’s short duration effect of 6 minutes is due to the rapid redistribution from the central nervous system to other tissues; however, it is dose dependent, and repeated dosing can lead to drug accumulation that prolongs sedation.13

Propofol typically is given in the emergency department as an initial bolus of 1 mg/kg followed by 0.5 mg/kg every 3 minutes until the desired level of sedation is achieved.13 General recommendations indicate a lower dose in patients older than 65 as they can achieve sedation more rapidly than younger patients.14

Common adverse effects include

undersedation, oversedation, hypoxemia, hypotension, and respiratory depression.15,16 Notably absent from its list of adverse effects are nausea, confusion, and agitation.12 The concurrent administration of opioids and propofol should be avoided due to a significant risk of respiratory depression. Propofol appears to be as safe and effective as midazolam, with the added benefit of a significantly shorter length of stay in the emergency department.17,18 When compared to ketamine, one study shows propofol has a decreased rate of respiratory depression and a shorter time to recovery, with no difference in the number of clinical interventions used for respiratory depression.19 Contraindications to its use include soy or egg allergies.13

KetofolKetofol is low-dose ketamine with

propofol, a combination designed to maximize the benefits and minimize the side effects of each drug. The two agents possess entirely different mechanisms of action and clinical features that may allow one drug to offset the other drug’s limitations. For example, propofol’s most feared side effects are cardiovascular

and respiratory depression, which might be mitigated by ketamine’s sympathomimetic properties and the ability to preserve airway reflexes.20 Ketamine also can provide analgesic effects that propofol lacks. Emesis and emergent reactions are ketamine’s principal adverse effects, which can be diminished by propofol’s antiemetic qualities.21

New evidence reinforces the efficacy and safety of ketofol over other agents. It is associated with less oxygen desaturation,22 less vomiting, and greater patient/provider satisfaction than the versed/fentanyl combination.22 Ketofol also appears to be similar in efficacy and respiratory depression to propofol alone, but results in more consistent sedation while using less propofol during the procedure.20 It is unclear if ketofol has fewer adverse effects than ketamine or propofol alone. The typical dose of ketofol for PSA is 1 mg/kg of ketamine combined with 1 mg/kg of propofol.

Fentanyl/Midazolam A combination of fentanyl

and midazolam traditionally has been used for procedural sedation and analgesia. Midazolam is a

Table 3.Contraindications for Ketamine9

Absolute

Age <3 months

Psychosis

Relative

Procedures stimulating posterior oropharynx

History of tracheal surgery or tracheal stenosis

Active pulmonary disease including URI or asthma

Significant cardiovascular disease (eg, angina, heart failure, hypertension)

Increased intracranial pressure (eg, intracranial mass, hydrocephalus)

Increased intraocular pressure (eg, glaucoma, acute globe injury)

Porphyria or thyroid disease

Page 15: Volume 29 • Number 8 In This Issue · with abdominal pain as a sign of acute gastroenteritis; however, appendicitis remains the most common, non-traumatic surgical emergency in

August 2015 • Volume 29 • Number 8

15

benzodiazepine that provides anxiolysis and amnesia without analgesic effects.23 Its lipophilic properties and ability to cross the blood-brain barrier result in quick onset times; however, repeated dosing can cause drug accumulation in the adipose tissue, leading to prolonged sedation. The typical dose for midazolam is 1-2 mg in adults and 0.05-0.1 mg/kg in children every 3-5 minutes.

Fentanyl, a synthetic opioid that is 75 to 125 times more potent than morphine, possesses analgesic properties without any amnestic effects. Fentanyl has an onset time of 2-3 minutes and a duration of 30 minutes.23 The typical regimen used in PSA is 0.05-0.1 mg/kg of midazolam with 0.5-1 mcg/kg of fentanyl. Although midazolam alone has not been shown to cause respiratory depression, the combined effects of midazolam and fentanyl can result in significant respiratory depression. To mitigate this complication, midazolam should be given first, followed by titrated fentanyl.

EtomidateEtomidate is a short-acting

hypnotic that acts on gamma-aminobutyric acid (GABA) receptors.24 It possesses sedative and amnestic properties without any analgesic effects and has many advantages, including quick onset (<1 minute), short duration of action (5-15 minutes), and minimal effects on

respiratory and cardiovascular function.23

The dose of etomidate for PSA is 0.15-0.2 mg/kg; it is usually given with an opioid such as fentanyl for analgesia. Research indicates that etomidate is safe and effective for procedural sedation and causes no major respiratory complications. Its potential side effects in PSA include myoclonus, adrenal insufficiency, and respiratory depression.25 Of these, myoclonus is the most common (reported in approximately 20% to 40% of patients); this complication

Pearls• Know the agents commonly used for PSA well, and be

familiar with their side effects and complications.

• The key to successful PSA is preparation; make sure all necessary materials are in place prior to starting any procedure.

• Titrate the medications to achieve the desired level of sedation, and allow an appropriate time gap before redosing.

• Anticipate and be prepared for complications should they arise.

Pitfalls• Failing to adequately assess the patient’s airway prior to sedation.

• Failing to have proper equipment and supplies readily available at the bedside, including reversal agents.

• Routinely using prophylactic benzodiazepine or anticholinergic with ketamine.

• Not anticipating or preparing for complications of sedation, including allergic reactions and respiratory or cardiac arrest.

• Failing to adequately monitor the patient after the procedure.

is thought to be dose related and can hinder procedure success rates.25

Adrenal insufficiency typically occurs with the continuous infusion of etomidate. Although decreased cortisol levels have been reported with a single dose of the drug, the clinical significance of such findings remains unclear.26 Respiratory depression occurs in approximately 10% of patients, with most cases resolving without major intervention or consequences.

DexmedetomidineDexmedetomidine is a relatively

new selective alpha-2 adrenoceptor agonist similar to clonidine. The medication provides sedation without respiratory depression, and decreases sympathetic tone while preserving psychomotor function.24 It typically is used in ICU settings as a short-term sedative in mechanically-ventilated patients or in cases of alcohol withdrawal, where it has been shown to reduce the required amounts of anesthetics and opioids.27

Procedural sedation exists in a continuum of

altered level of consciousness ranging from

minimal to general anesthesia, with dissociative

sedation falling outside of the continuum.

Page 16: Volume 29 • Number 8 In This Issue · with abdominal pain as a sign of acute gastroenteritis; however, appendicitis remains the most common, non-traumatic surgical emergency in

Critical Decisions in Emergency Medicine

16

The medication’s most common side effects are hypotension, brady -cardia, and dry mouth.24 The recommended dose for PSA is a 0.5-1 mcg/kg loading dose over 10 minutes, followed by 0.2-1 mcg/kg continuous infusion. There is limited data on the use of dexmedetomidine for PSA in the emergency department. Due to hepatic metabolism and renal clearance of the drug, dose reduction should be considered in elderly patients (>65 years) or in cases of hepatic or renal impairment.

CRITICAL DECISIONWhat clinical personnel are required when performing procedural sedation?

All personnel performing PSA must have a thorough understanding of the medications, the ability to

properly monitor the patient, and training in the management of potential complications.3 Guidelines vary by institution, but procedural sedation and analgesia typically requires two trained practitioners – one to monitor the patient (eg, a nurse), and one to perform the procedure (a physician).1,24

In some instances, the emergency physician might be able to do both tasks. This can be possible, for instance, if the physician is able to maintain visual or verbal communication with the patient while performing the procedure. Some institutions, however, require at least two emergency physicians to be present for PSA, one of whom must be a board-certified faculty member.

CRITICAL DECISIONHow should patients be monitored during PSA, and what equipment is necessary?

Proper monitoring of the patient during PSA is critical and should involve both clinical assessments and objective parameters. Clinical assessments usually are performed using the Ramsay sedation or observer’s assessment of alertness/sedation scales. Objective measures include vital signs (blood pressure, heart rate, and respiratory rate), which should be obtained at regular intervals before, during, and after the procedure.1,4

Pulse oximetry and cardiac rhythm should be monitored continuously.3 The limitation of pulse oximetry is its inability to identify

Table 4.Medications Used in PSA7,9,23

Drug Route Dose Repeat Dose Onset of Action Duration

Dexmedetomidine IV 0.5-1 mcg/kg over 10 min, continuous infusion of 0.2-1 mcg/kg/hr

10 min 1-2 hrs

Etomidate IV 0.2-0.3 mg/kg 0.05 mg/kg q3-5 min

<1 min 5-15 min

Ketamine IV 1-2 mg/kg 0.25-0.5 mg/kg q5-10 min

<1 min 10-20 min

IM 4-5 mg/kg 2-4 mg/kg q10-15 min

2-5 min 15-30 min

Propofol IV 1 mg/kg 0.5 mg/kgq3-5 min

30-45 sec Dose dependent6 min

Ketofol IV 1 mg/kg ketamine: 1 mg/kg propofol

Fentanyl IV 0.5-1 mcg/kg slow IV push

0.5 mcg/kg q2 min

2-3 min 30 min

Midazolam

IV 1-2 mg (adults)0.05-0.1 mg/kg (children)

q3-5 min 1-2 min 30 min

Page 17: Volume 29 • Number 8 In This Issue · with abdominal pain as a sign of acute gastroenteritis; however, appendicitis remains the most common, non-traumatic surgical emergency in

August 2015 • Volume 29 • Number 8

17

early problems with ventilation and detect hypercarbia, especially if supplemental oxygen is used during PSA. Capnography provides a way of monitoring the adequacy of ventilation and identifying potential respiratory compromise before oxygen desaturation takes place.28

The American Society of Anesthesiologists recommends the use of continuous capnography for moderate and deep sedation in non-operative settings.3 Although its use allows early detection of hypoventilation, there is no evidence that this feature has any impact on clinical outcomes.1 Current ACEP guidelines do not require the use of capnography for PSA; in many institutions, its use is left to the discretion of the emergency physician.2

Equipment and SuppliesProper equipment and supplies

must be in place to manage the potential complications of procedural sedation and analgesia, including allergic reactions, cardiac arrest, and respiratory arrest.1 Oxygen, suction, bag-valve masks, intubation equipment, and advanced cardiac life-support medications should be readily available during the procedure.3,4 If opioids or benzodiazepines are used, reversal agents, naloxone, and flumazenil, should be easily accessible.3

CRITICAL DECISIONWhen is it safe to discharge a patient home following PSA?

Patients should be observed and monitored until they are no longer at risk of respiratory depression. They should be alert, oriented, and back to their baseline level of consciousness prior to discharge.3 If any reversal agents are used, monitor the patient for up to 2 hours after the medication is given. There is no evidence that supports the pre-discharge requirement of a patient being able to tolerate oral intake and ambulate without assistance.

Clear discharge instructions should be given and explained to the patient and the caregiver. A friend or caregiver should be available to monitor the patient for several hours following discharge. Although serious adverse events such as hypoxia rarely occur after PSA, patients can experience mild symptoms such as nausea, fatigue, and lightheadedness for up to 24 hours.

Case Resolution

■ Case 1The clinical team concluded that

establishing IV access would be difficult and distressful for the child with a forehead laceration, so he was given 5 mg/kg of IM ketamine. The laceration, which revealed a tear in the galea, was thoroughly irrigated and repaired without complications. The boy had one episode of emesis after the procedure. He was observed for another 20 minutes and discharged under the care of his parents.

■ Case 2The patient with atrial fibrillation

required emergent cardioversion; he was placed on a cardiac monitor, and supplemental oxygen and IV fluids were started. Due to its hemodynamic stability and minimal cardiovascular effects, etomidate was chosen as a sedative agent with fentanyl for analgesia. The patient underwent successful cardioversion to sinus rhythm. His blood pressure improved and the symptoms of lightheadedness and shortness of breath resolved. He was admitted to medical services for further evaluation of his new-onset atrial fibrillation, and remained in sinus rhythm upon transfer to the floor.

■ Case 3Radiographs were obtained of the

elderly car accident victim’s chest, head, pelvis, and hips. The hip x-ray showed a posterior dislocation without fracture; head and cervical

spine radiographs did not show any abnormalities. The patient was placed on a cardiac monitor, supplemental oxygen was given, and an IV was established. He underwent PSA using low-dose propofol as a sedative agent and fentanyl for analgesia. Several attempts at hip reduction were made without success; the patient needed repeated doses of propofol to achieve sedation. His blood pressure dropped to 104/60 during the procedure, and he subsequently was taken to the operating room for open reduction by an orthopedic surgeon.

Summary of LessonProcedural sedation and analgesia

is becoming a common practice in the emergency department for a number of procedures. Emergency physicians must select the sedative agent or combination of agents appropriate for the type of procedure and level of sedation required. Providers must be knowledgeable about the agents commonly used in PSA, and skilled in identifying and managing any complications that might arise. Emergency physicians also must be familiar with the institutional and published guidelines regarding personnel, monitoring, and discharge criteria after PSA.

References1. Godwin SA, Caro DA, Wolf SJ, et al. Clinical policy:

procedural sedation and analgesia in the emergency department. Ann Emerg Med. 2005;45(2):177–196.

2. Godwin SA, Burton JH, Gerardo CJ, et al. American College of Emergency Physicians. ACEP clinical policy: procedural sedation and analgesia in the emergency department. Ann Emerg Med. 2014;63:247-258.

3. American Society of Anesthesiologists Task Force on Sedation and Analgesia by Non-Anesthesiologists. Practice guidelines for sedation and analgesia by non-anesthesiologists. Anesthesiology. 2002;96(4):1004–1017.

4. Coté CJ, Wilson S; American Academy of Pediatrics, et al. Guidelines on monitoring and management of pediatric patients during and after sedation for diagnostic and therapeutic procedures: an update. Pediatrics. 2006;118(6):2587–2596.

5. O’Connor RE, Sama A, Burton JH, et al. Procedural sedation and analgesia in the emergency department: recommendations for physician credentialing, privileging, and practice. Ann Emerg Med. 2011;58(4):365–370.

6. Mace SE, Brown LA, Francis L, et al. Clinical policy: Critical issues in the sedation of pediatric patients in the emergency department. Ann Emerg Med. 2008;51(4):378–391.

7. Madati PJ. Ketamine: pediatric procedural sedation in the emergency department. Pediatr Emerg Med Pract. 2011;8(1):1–16.

Page 18: Volume 29 • Number 8 In This Issue · with abdominal pain as a sign of acute gastroenteritis; however, appendicitis remains the most common, non-traumatic surgical emergency in

Critical Decisions in Emergency Medicine

18

8. Bahn EL, Holt KR. Procedural sedation and analgesia: a review and new concepts. Emerg Med Clin North Am. 2005;23(2):503–517. .

9. Green SM, Roback MG, Kennedy RM, Krauss B. Clinical practice guideline for emergency department ketamine dissociative sedation: 2011 update. Ann Emerg Med. 2011;57(5):449–461.

10. Strayer RJ, Nelson LS. Adverse events associated with ketamine for procedural sedation in adults. Am J Emerg Med. 2008;26(9):985–1028.

11. Green SM, Roback MG, Krauss B, et al. Predictors of emesis and recovery agitation with emergency department ketamine sedation: an individual-patient data meta-analysis of 8,282 children. Ann Emerg Med. 2009:54(2):171-180.e174

12. Burton JH, Miner JR, Shipley ER, et al. Propofol for emergency department procedural sedation and analgesia: a tale of three centers. Acad Emerg Med. 2006;13(1):24-30.

13. Miner JR, Burton JH. Clinical practice advisory: Emergency department procedural sedation with propofol. Ann Emerg Med. 2007;50(2):182–187.

14. Weaver CS, Terrell KM, Bassett R, et al. ED procedural sedation of elderly patients: is it safe? Am J Emerg Med. 2011;29(5):541–544.

15. Weaver CS, Hauter WE, Brizendine EJ, Cordell WH. Emergency department procedural sedation with propofol: is it safe? J Emerg Med. 2007;33(4):355–361.

16. McGrane O, Hopkins G, Nielson A, Kang C. Procedural sedation with propofol: a retrospective review of the experiences of an emergency medicine residency program 2005 to 2010. Am J Emerg Med. 2012;30(5):706-711.

17. Dunn T, Mossop D, Newton A, Gammon A. Propofol for procedural sedation in the emergency department. Emerg Med J. 2007;24(7):459–461.

18. Rahman NH, Hashim A. The use of propofol for procedural sedation and analgesia in the emergency department: a comparison with midazolam. Emerg Med J. 2011;28(10):861–865.

19. Miner JR, Gray RO, Bahr J, et al. Randomized clinical trial of propofol versus ketamine for procedural sedation in the emergency department. Acad Emerg Med. 2010;17(6):604-611.

20. David H, Shipp J. A randomized controlled trial of ketamine/propofol versus propofol alone for emergency department procedural sedation. Ann Emerg Med. 2011;57(5):435–441.

21. Miner JR, Danahy M, Moch A, Biros M. Randomized clinical trial of etomidate versus propofol for procedural sedation in the emergency department. Ann Emerg Med. 2007;49(1):15–22.

22. Nejati A, Moharari RS, Ashraf H, et al. Ketamine/propofol versus midazolam/fentanyl for procedural sedation and analgesia in the emergency department: a randomized, prospective, double-blind trial. Acad Emerg Med. 2011;18(8):800–806.

23. Mace SE, Barata IA, Cravero JP, et al. Clinical policy: evidence-based approach to pharmacologic agents used in pediatric sedation and analgesia in the emergency department. Ann Emerg Med. 2004;44(4):342–354.

24. Tobias JD, Leder M. Procedural sedation: A review of sedative agents, monitoring, and management of complications. Saudi J Anaesth. 2011;5(4):295-410.

25. Green SM, Andolfatto G, Krauss BS. Ketofol for procedural sedation? pro and con. Ann Emerg Med. 2011;57(5):444-448.

26. Schenarts CL, Burton JH, Riker RR. Adrenocortical dysfunction following etomidate induction in emergency department patients. Acad Emerg Med. 2001;8:1-7

27. Hoy SM, Keaing GM. Dexmedetomidine: a review of its use in mechanically ventilated patients in intensive care settings and for procedural sedation. Drugs. 2011;71(11):1481-501.

28. Krauss B, Hess DR. Capnography for procedural sedation and analgesia in the emergency department. Ann Emerg Med. 2007;50(2):172–181.

Page 19: Volume 29 • Number 8 In This Issue · with abdominal pain as a sign of acute gastroenteritis; however, appendicitis remains the most common, non-traumatic surgical emergency in

August 2015 • Volume 29 • Number 8

19

The Critical Image

Upon arrival at the patient’s home, EMS noted the patient to be hypertensive and hypoxic to 70% on room air. On arrival to the emergency department, her vital signs are blood pressure 234/128, heart rate 115, respiratory rate 36, and oxygen saturation 100% on 100% non-rebreather mask. The physical examination reveals tachypnea, jugular venous distension, and bilateral wheezing without leg edema. The cardiac examination is regular, but the patient is tachycardic without accessory heart sounds. An ECG shows sinus tachycardia without ST elevation or depression.

The patient is placed on an intravenous nitroglycerin infusion, and noninvasive positive-pressure ventilation is initiated. Given her mixed examination features, which suggest both heart failure and possible COPD exacerbation, albuterol and corticosteroids also are administered. A portable chest radiograph is performed.

■ Case

A 52-year-old woman presents with acute respiratory distress while at rest and when smoking cigarettes. She denies chest pain, but admits to using crack cocaine several hours earlier. Her medical history includes heart failure with an ejection fraction of 30%, ST-segment elevation myocardial infarction, COPD, hypertension, chronic descending aortic dissection, and stroke.

A. Portable AP chest radiograph on emergency department presentation, which shows classic signs of pulmonary edema, including cephalization, fluid in the minor fissure, and dependent interstitial edema.

Increased vascular prominence in upper lung zones (cephalization)

Fluid in minor fissure

Increased basilar interstitial prominence

{{

A

Page 20: Volume 29 • Number 8 In This Issue · with abdominal pain as a sign of acute gastroenteritis; however, appendicitis remains the most common, non-traumatic surgical emergency in

Critical Decisions in Emergency Medicine

20

B. Baseline PA chest radiograph of the same patient following a hospitalization for flash pulmonary edema. At the time of this x-ray, she had a normal blood pressure and no pulmonary symptoms, and had undergone diuresis in the hospital. The chest radiograph shows less interstitial prominence compared with the radiograph obtained during the acute presentation. The minor fissure contains less fluid and is only faintly visible.

B

Decreased vascular prominence in upper lung zones (normal)

Minor fissure less prominent

Decreased basilar interstitial prominence

{{

The Critical Image

Key Points

• Normally, gravity causes dependent pulmonary blood vessels to fill and appear most prominently, while vessels in the upper lung zones are relatively empty and radiographically inconspicuous. In cases of abnormally high pulmonary vascular pressures and volume, vessels in the upper lung zones also appear full, a finding termed “cephalization.” As pulmonary edema progresses, dependent areas develop increased interstitial density and fluid can accumulate in the fissures and dependent pleural spaces. If pulmonary edema is severe, fluid can accumulate in alveoli, resulting in air bronchograms, the silhouetting of air-filled bronchi by adjacent fluid-filled alveoli (not seen in this case). Alveolar fluid sometimes obscures landmarks such as the right and left heart borders and diaphragm, resulting in loss of the normal “silhouette sign.” The normal silhouette sign occurs when tissues of different density (eg lung and heart) abut each other, creating a distinct radiographic border or silhouette. When two tissues of similar density (eg fluid-laden lung and heart) are contiguous, no border can be seen between them.1

• Comparison of current images with prior images can be essential to understanding a patient’s acute presentation. Note that a patient might be suffering from the same type of acute medical emergency they were experiencing when the previous x-rays were taken. Correlating the prior image with the prior clinical presentation is crucial to avoid mistaking “unchanged” for “chronic” or “normal.” In the patient described above, a comparison of images A and C reveals little change because both x-rays were acquired during episodes of acute pulmonary edema.

Page 21: Volume 29 • Number 8 In This Issue · with abdominal pain as a sign of acute gastroenteritis; however, appendicitis remains the most common, non-traumatic surgical emergency in

August 2015 • Volume 29 • Number 8

21

C

Fluid in minor fissure

Increased basilar interstitial prominence {

C. Portable AP chest radiograph from a previous emergency department presentation for hypertension and acute pulmonary edema. Note the similarities to A.

Case ResolutionEnalaprilat was administered intravenously, and the patient rapidly improved. The nitroglycerin infusion and BiPap were discontinued within 1 hour, and she was admitted for further care.

Feature Editor: Joshua S. Broder, MD, FACEP. See also Diagnostic Imaging for the Emergency Physician by Dr. Broder, available from the ACEP Bookstore, www.acep.org/bookstore.

Case Contributor: Hirsh Sandesara, MD Cases for “The Critical Image” are contributed by the residents and faculty of the Duke University Hospital Emergency Medicine Residency.

References1. Felson B, Felson H. Localization of intrathoracic lesions by means of the postero-anterior roentgenogram; the silhouette sign. Radiology 1950;55:363–74.

Page 22: Volume 29 • Number 8 In This Issue · with abdominal pain as a sign of acute gastroenteritis; however, appendicitis remains the most common, non-traumatic surgical emergency in

Critical Decisions in Emergency Medicine

22

Qualified, paid subscribers to Critical Decisions in Emergency Medicine may receive CME certificates for up to 5 ACEP Category I credits, 5 AMA PRA Category 1 Credits™, and 5 AOA Category 2-B credits for answering the following questions. To receive your certificate, go to www.acep.org/newcriticaldecisionstesting and submit your answers online. On achieving a score of 75% or better, you will receive a printable CME certificate. You may submit the answers to these questions at any time within 3 years of the publication date. You will be given appropriate credit for all tests you complete and submit within this time. Answers to this month’s questions will be published in next month’s issue.

CME Questions

1. Which of the following makes it challenging to diagnose the cause of abdominal pathology in children?A. Abdominal pain in an uncommon complaint in

pediatric patientsB. Children often are not as cooperative with the

physical examination and history taking as adult patients

C. Intra-abdominal pathology is very common in cases of chronic abdominal pain

D. Pediatric patients almost always need to be sedated for CT scans

2. Which imaging study should be the first line in the emergency department evaluation of a patient with potential intussusception?A. Abdominal radiographB. Air-contrast enemaC. CT scan of the abdomenD. Ultrasound

3. Which of the following describes why a CT scan can be more valuable than ultrasound in the evaluation of a patient with suspected appendicitis?A. CT scans provide a faster diagnostic speed B. Surgeons prefer CT scan imagingC. Ultrasound is hampered by low sensitivity/specificityD. Ultrasound often yields inadequate or non-diagnostic

results

4. Which of the following is true regarding abdominal CT scans in cases of blunt abdominal trauma?A. CT scans are the only way to avoid missed injuries

in pediatric patients who can’t talk or have had an unreliable examination

B. CT scans have become less popular with the emergence of new technologies

C. CT scans should be obtained in all patients with abdominal trauma

D. CT scans should be reserved for patients deemed at highest risk for intra-abdominal injury

5. What is the most appropriate radiologic study in a pediatric patient with examination findings suggestive of classic appendicitis?A. Abdominal CT scanB. Abdominal radiographC. Abdominal ultrasoundD. No imaging is required

6. Which is true of the use of bedside ultrasound for FAST examinations in pediatric trauma patients?A. Its efficacy has been validated in children, just as in

adult trauma patientsB. It is as sensitive as a CT scan for picking up intra-

abdominal injuryC. Its use and popularity are increasingD. The success of the FAST examination is operator

independent

7. Which of the following is the most common malignancy that presents as an abdominal mass in infants or children?A. Germ cell tumorB. LymphomaC. NeuroblastomaD. Wilms tumor

8. How can a CT scan be justified and used in the case of a suspected abdominal mass?A. A CT scan is never indicated in the setting of an

abdominal mass B. To characterize the origin of the mass, extent of the

disease, and identify sites of metastasisC. To determine bony involvement, if present D. To determine the specific components (cystic vs.

solid) of the mass

9. Which of the following imaging studies should be the first line in the initial evaluation of chronic abdominal pain with obstructive symptoms or dyspepsia?A. Abdominal radiographB. Air-contrast enemaC. CT scan of the abdomenD. Upper GI with small bowel follow through

Page 23: Volume 29 • Number 8 In This Issue · with abdominal pain as a sign of acute gastroenteritis; however, appendicitis remains the most common, non-traumatic surgical emergency in

August 2015 • Volume 29 • Number 8

23

Answer Key for July 2015, Volume 29, Number 7

1 2 3 4 5 6 7 8 9 10 11 12 13 14 15 16 17 18 19 20B A B B B D A A A B D D B C B A C C D A

10. Which of the following tests will yield a definitive diagnosis of ectopic pregnancy in almost all cases?A. Serum quantitative beta-hCG and pelvic examinationB. Serum quantitative beta-hCG and transvaginal

ultrasoundC. Transvaginal ultrasound D. Urine beta-hCG

11. Which of the following accurately describes moderate sedation?A. A state of depressed consciousness during which

patients cannot be easily aroused, but respond purposefully to repeated or painful stimuli; possible impairment in ventilation

B. A state of depressed consciousness during which patients respond normally to verbal commands and light stimulation; ventilatory and cardiovascular functions remain unaffected

C. Drug-induced depression of consciousness during which patients respond purposefully to verbal commands or light stimulation, while spontaneous ventilation and cardiovascular function are maintained

D. Trance-like cataleptic state of profound analgesia and amnesia with maintenance of airway reflexes, spontaneous respirations, and cardiopulmonary stability

12. Which of the following is an absolute contraindication to ketamine use?A. Acute globe injuryB. Age less than 3 monthsC. Coronary artery diseaseD. Head trauma

13. Which of the following side effects is not associated with ketamine?A. BradycardiaB. BronchodilationC. Emergent reactionD. Hypersalivation

14. Which of the following is associated with the IM route of administration for ketamine?A. Higher rate of emergent reactionsB. Higher rate of emesisC. Higher rate of hypersalivationD. Higher rate of respiratory complications

15. Which of the following is not included in ACEP’s guidelines for standard monitoring of PSA patients?A. Blood pressure measurementB. CapnographyC. Cardiac monitorD. Pulse oximetry

16. In which patient should PSA be performed without consideration of the recommended fasting time?A. A child with an abscess on the thigh who ate a meal

1 hour prior to arrivalB. An alcoholic man with a complex lip laceration who

ate a meal 1 hour prior to arrivalC. An elderly woman with a displaced forearm fracture

with neurovascular compromise who ate a meal 1 hour prior to arrival

D. An obese man with an elbow dislocation who ate a meal 1 hour prior to arrival

17. Which of the following reflects the recommended fasting time for breast milk, according to ASA fasting guidelines?A. 1 hourB. 2 hoursC. 4 hoursD. 6 hours

18. Which scenario is inappropriate for the dose reduction of dexmedetomidine?A. Cardiac diseaseB. Liver diseaseC. Old ageD. Renal insufficiency

19. Which is not a side effect of etomidate?A. Adrenal insufficiencyB. HypotensionC. MyoclonusD. Respiratory depression

20. Which protocol describes a method for mitigating the respiratory effects of opioid/benzodiazepine?A. Give benzodiazepine prior to opioidB. Give opioid prior to benzodiazepineC. Give both opioid and benzodiazepine every 30

secondsD. Give both opioid and benzodiazepine together

Page 24: Volume 29 • Number 8 In This Issue · with abdominal pain as a sign of acute gastroenteritis; however, appendicitis remains the most common, non-traumatic surgical emergency in

NONPROFIT

U.S. POSTAGE

P A I DDALLAS, TX

PERMIT NO. 1586

August 2015 • Volume 29 • Number 8Critical Decisions in Emergency Medicine is the official CME publication of the American College of Emergency Physicians. Additional volumes are available to keep emergency medicine professionals up-to-date on relevant clinical issues.

Editor-in-ChiefLouis G. Graff IV, MD, FACEP Professor of Traumatology and Emergency Medicine, Professor of Clinical Medicine, University of Connecticut School of Medicine; Farmington, CT

Section EditorJ. Stephen Bohan, MS, MD, FACEP Executive Vice Chairman and Clinical Director, Department of Emergency Medicine, Brigham & Women’s Hospital; Instructor, Harvard Medical School, Boston, MA

Feature EditorsMichael S. Beeson, MD, MBA, FACEP Program Director, Department of Emergency Medicine, Akron General, Akron, Ohio; Professor, Clinical Emergency Medicine, Northeastern Ohio Universities College of Medicine, Rootstown, OH

Joshua S. Broder, MD, FACEP Associate Clinical Professor of Surgery, Residency Program Director, Division of Emergency Medicine, Duke University Medical Center, Durham, NC

Amal Mattu, MD, FACEP Professor and Vice Chair, Department of Emergency Medicine, Director, Faculty Development and Emergency Cardiology Fellowships, University of Maryland School of Medicine, Baltimore, MD

Associate EditorsWalter L. Green, MD, FACEP Assistant Emergency Medicine Program Director, Associate Professor of Emergency Medicine, University of Texas Southwestern Medical Center, Dallas, TX

Daniel A. Handel, MD, MPH, FACEP Chief Medical Officer, Associate Professor of Medicine and Pediatrics, Division of Emergency Medicine, Medical University of South Carolina, Charleston, SC

Frank LoVecchio, DO, MPH, FACEP Research Director, Maricopa Medical Center Emergency Medicine Program; Medical Director, Banner Poison Control Center, Phoenix, Arizona; Professor, Midwestern University/Arizona College of Osteopathic Medicine, Glendale, AZ

Sharon E. Mace, MD, FACEP Associate Professor, Department of Emergency Medicine, Ohio State University School of Medicine; Faculty, MetroHealth Medical Center/Cleveland Clinic Foundation Emergency Medicine Residency Program; Director, Pediatric Education/Quality Improvement and Observation Unit, Cleveland Clinic Foundation, Cleveland, OH

Lynn P. Roppolo, MD, FACEP Associate Emergency Medicine Residency Director, Associate Professor of Emergency Medicine, University of Texas Southwestern Medical Center, Dallas, TX

Robert A. Rosen, MD, FACEP Medical Director, Culpeper Regional Hospital, Culpeper, Virginia; Associate Professor, Emergency Medicine, University of Virginia School of Medicine, Charlottesville, VA

Robert C. Solomon, MD, FACEP Core Faculty, Emergency Medicine Residency, Allegheny General Hospital, Pittsburgh, Pennsylvania; Assistant Professor (adjunct) Emergency Medicine, Temple University School of Medicine, Philadelphia, PA

George Sternbach, MD, FACEP Clinical Professor of Surgery (Emergency Medicine), Stanford University Medical Center, Stanford, CA

Kathleen Wittels, MD Instructor, Harvard Medical School, Boston, MA

Steven J. Warrington, MD Institutional Director of Simulation; Core Faculty, Emergency Medicine and General Surgery Residency Programs; Associate Medical Director for Emergency Department Education and Outreach, Kaweah Delta Medical Center, Visalia, CA

Editorial StaffRachel Donihoo, Managing EditorJessica Hamilton, Educational Products AssistantLexi Schwartz, Subscriptions CoordinatorMarta Foster, Director, Educational Products

Critical Decisions in Emergency Medicine is a trademark owned and published monthly by the American College of Emergency Physicians, PO Box 619911, Dallas, TX 75261-9911. Send address changes and comments to Critical Decisions in Emergency Medicine, PO Box 619911, Dallas, TX 75261-9911, or to [email protected]; call toll-free 800-798-1822, or 972-550-0911.Copyright 2015 © by the American College of Emergency Physicians. All rights reserved. No part of this publication may be reproduced, stored, or transmitted in any form or by any means, electronic or mechanical, including storage and retrieval systems, without permission in writing from the Publisher. Printed in the USA.The American College of Emergency Physicians (ACEP) makes every effort to ensure that contributors to its publications are knowledgeable subject matter experts. Readers are nevertheless advised that the statements and opinions expressed in this publication are provided as the contributors’ recommendations at the time of publication and should not be construed as official College policy. ACEP recognizes the complexity of emergency medicine and makes no representation that this publication serves as an authoritative resource for the prevention, diagnosis, treatment, or intervention for any medical condition, nor should it be the basis for the definition of, or standard of care that should be practiced by all health care providers at any particular time or place. Drugs are generally referred to by generic names. In some instances, brand names are added for easier recognition. Device manufacturer information is provided according to style conventions of the American Medical Association. ACEP received no commercial support for this publication. To the fullest extent permitted by law, and without limitation, ACEP expressly disclaims all liability for errors or omissions contained within this publication, and for damages of any kind or nature, arising out of use, reference to, reliance on, or performance of such information.

ISSN2325-0186(Print) ISSN2325-8365(Online)

Feature Editors: Michael S. Beeson, MD, MBA, FACEP; Steven Warrington, MD

The Drug BoxMethotrexateBy Elizabeth Ulrich, DO, Akron General Medical Center, Akron, Ohio

Methotrexate is an antifolate metabolite that inhibits DNA synthesis, cellular replication, and repair. The medication has multiple applications, but primarily is used for the treatment of autoimmune disorders and as a chemotherapeutic agent. It is classified as an antineoplastic, antimetabolite, immunosuppressive, and disease-modifying anti-rheumatic drug (DMARD). Early, aggressive treatment of rheumatoid arthritis improves pain and swelling, while further reducing joint damage. Routine monitoring of CBC, creatinine, and LFTs are recommended every 2 months.

Mechanism of Action

Methotrexate irreversibly binds dihydrofolate reductase, ultimately inhibiting purine and thymidylic acid synthesis. This leads to impaired DNA synthesis and repair, which inhibits cellular replication. Actively proliferative tissues are most susceptible to methotrexate.

Indications Treatment of several types of cancer (eg, lymphoma, leukemia, breast, skin, head, neck, and lung), rheumatoid arthritis, psoriasis, juvenile idiopathic arthritisOff-label: Crohn disease, dermatomyositis, graft vs. host, ectopic pregnancy

Dosing (ADULTS)

7.5 mg once weekly OR 2.5 mg every 12 hours for 3 doses/weekCancer dosing varies based on type of treatment. Ranges include 30–40 mg/m2/week to 100–12,000 mg/m2 with leucovorin rescue. Note: Pediatric dosing varies depending on type of cancer or autoimmune disease. Administration: PO, IM, SQ, IV, intrathecalRenal insufficiency: Clcr 10-50 ml/min — reduced dose to 50%; Clcr<10 ml/min — avoid use

Side Effects Multiple US boxed warnings: Associated with transaminitis, fibrosis, cirrhosis, hepatotoxicity, renal damage, pneumonitis, malignant lymphoma, diarrhea, ulcerative stomatitis, TENS, SJS, and bone-marrow suppression, including aplastic anemia. Use caution with NSAIDS. Medication is teratogenic; caution for opportunistic infections. There are many other side effects, ranging from skin photosensitivity to pulmonary fibrosis.

Precautions Contraindications: Hypersensitivity to methotrexate, alcoholism, liver disease, immunodeficiency, and blood dyscrasias Pregnancy category: X Lactation: Contraindicated